Range of $ y = frac{x^3+3x^2+7x-11}{x^2+5x-6} $?












0












$begingroup$


How do I find the range of $ y = frac{x^3+3x^2+7x-11}{x^2+5x-6} $? Would someone please help me understand how to do this by the method of forming a quadratic equation in x?
A video lesson I was watching first factorises the numerator and denominator and finds that (x-1) is a common factor. It then forms a quadratic equation in x and finds values of y for discriminant greater than or equal to zero, getting a range R. It removes the (x-1) factor from numerator and denominator. then substitutes x=1 to get a y value 16/7. I don't get why this step works. It then equates 16/7 to the original expression to find x values,one of which is one and one of which is -19/7. It concludes that range is finally R. There is also a verification step which doesn't apply to this question but to other similar questions in which we try to get coefficient of $x^2 = 0$ and then check that y value. I don't get the purpose of any of this.
I tried going through this question but it seems to deal with complex numbers and doesn't answer my doubts. What is the range of $f(x) = frac{x^2-5x+6}{x-3}$?










share|cite|improve this question









$endgroup$








  • 2




    $begingroup$
    One thing to consider, since the numerator is a higher degree polynomial than the denominator, is to divide the denominator into the numerator. This is discussed, with a few good answers with detailed explanations, in Polynomial Long Division Confusion (simplifying $frac{x^{5}}{x^{2}+1}$). However, keep in mind that any values of $x$ which cause the denominator to be $0$ need to be handled appropriately.
    $endgroup$
    – John Omielan
    Jan 13 at 2:07








  • 1




    $begingroup$
    What is the result after dividing the numerator and demoninator by x - 1? Clearly 1 is not in the domain of y so calcuatling the nonexistent y(1) is TVidiotcy. At best it could be a value of a continuous extension of y.
    $endgroup$
    – William Elliot
    Jan 13 at 3:33


















0












$begingroup$


How do I find the range of $ y = frac{x^3+3x^2+7x-11}{x^2+5x-6} $? Would someone please help me understand how to do this by the method of forming a quadratic equation in x?
A video lesson I was watching first factorises the numerator and denominator and finds that (x-1) is a common factor. It then forms a quadratic equation in x and finds values of y for discriminant greater than or equal to zero, getting a range R. It removes the (x-1) factor from numerator and denominator. then substitutes x=1 to get a y value 16/7. I don't get why this step works. It then equates 16/7 to the original expression to find x values,one of which is one and one of which is -19/7. It concludes that range is finally R. There is also a verification step which doesn't apply to this question but to other similar questions in which we try to get coefficient of $x^2 = 0$ and then check that y value. I don't get the purpose of any of this.
I tried going through this question but it seems to deal with complex numbers and doesn't answer my doubts. What is the range of $f(x) = frac{x^2-5x+6}{x-3}$?










share|cite|improve this question









$endgroup$








  • 2




    $begingroup$
    One thing to consider, since the numerator is a higher degree polynomial than the denominator, is to divide the denominator into the numerator. This is discussed, with a few good answers with detailed explanations, in Polynomial Long Division Confusion (simplifying $frac{x^{5}}{x^{2}+1}$). However, keep in mind that any values of $x$ which cause the denominator to be $0$ need to be handled appropriately.
    $endgroup$
    – John Omielan
    Jan 13 at 2:07








  • 1




    $begingroup$
    What is the result after dividing the numerator and demoninator by x - 1? Clearly 1 is not in the domain of y so calcuatling the nonexistent y(1) is TVidiotcy. At best it could be a value of a continuous extension of y.
    $endgroup$
    – William Elliot
    Jan 13 at 3:33
















0












0








0





$begingroup$


How do I find the range of $ y = frac{x^3+3x^2+7x-11}{x^2+5x-6} $? Would someone please help me understand how to do this by the method of forming a quadratic equation in x?
A video lesson I was watching first factorises the numerator and denominator and finds that (x-1) is a common factor. It then forms a quadratic equation in x and finds values of y for discriminant greater than or equal to zero, getting a range R. It removes the (x-1) factor from numerator and denominator. then substitutes x=1 to get a y value 16/7. I don't get why this step works. It then equates 16/7 to the original expression to find x values,one of which is one and one of which is -19/7. It concludes that range is finally R. There is also a verification step which doesn't apply to this question but to other similar questions in which we try to get coefficient of $x^2 = 0$ and then check that y value. I don't get the purpose of any of this.
I tried going through this question but it seems to deal with complex numbers and doesn't answer my doubts. What is the range of $f(x) = frac{x^2-5x+6}{x-3}$?










share|cite|improve this question









$endgroup$




How do I find the range of $ y = frac{x^3+3x^2+7x-11}{x^2+5x-6} $? Would someone please help me understand how to do this by the method of forming a quadratic equation in x?
A video lesson I was watching first factorises the numerator and denominator and finds that (x-1) is a common factor. It then forms a quadratic equation in x and finds values of y for discriminant greater than or equal to zero, getting a range R. It removes the (x-1) factor from numerator and denominator. then substitutes x=1 to get a y value 16/7. I don't get why this step works. It then equates 16/7 to the original expression to find x values,one of which is one and one of which is -19/7. It concludes that range is finally R. There is also a verification step which doesn't apply to this question but to other similar questions in which we try to get coefficient of $x^2 = 0$ and then check that y value. I don't get the purpose of any of this.
I tried going through this question but it seems to deal with complex numbers and doesn't answer my doubts. What is the range of $f(x) = frac{x^2-5x+6}{x-3}$?







algebra-precalculus functions polynomials






share|cite|improve this question













share|cite|improve this question











share|cite|improve this question




share|cite|improve this question










asked Jan 13 at 2:01









HemaHema

5791113




5791113








  • 2




    $begingroup$
    One thing to consider, since the numerator is a higher degree polynomial than the denominator, is to divide the denominator into the numerator. This is discussed, with a few good answers with detailed explanations, in Polynomial Long Division Confusion (simplifying $frac{x^{5}}{x^{2}+1}$). However, keep in mind that any values of $x$ which cause the denominator to be $0$ need to be handled appropriately.
    $endgroup$
    – John Omielan
    Jan 13 at 2:07








  • 1




    $begingroup$
    What is the result after dividing the numerator and demoninator by x - 1? Clearly 1 is not in the domain of y so calcuatling the nonexistent y(1) is TVidiotcy. At best it could be a value of a continuous extension of y.
    $endgroup$
    – William Elliot
    Jan 13 at 3:33
















  • 2




    $begingroup$
    One thing to consider, since the numerator is a higher degree polynomial than the denominator, is to divide the denominator into the numerator. This is discussed, with a few good answers with detailed explanations, in Polynomial Long Division Confusion (simplifying $frac{x^{5}}{x^{2}+1}$). However, keep in mind that any values of $x$ which cause the denominator to be $0$ need to be handled appropriately.
    $endgroup$
    – John Omielan
    Jan 13 at 2:07








  • 1




    $begingroup$
    What is the result after dividing the numerator and demoninator by x - 1? Clearly 1 is not in the domain of y so calcuatling the nonexistent y(1) is TVidiotcy. At best it could be a value of a continuous extension of y.
    $endgroup$
    – William Elliot
    Jan 13 at 3:33










2




2




$begingroup$
One thing to consider, since the numerator is a higher degree polynomial than the denominator, is to divide the denominator into the numerator. This is discussed, with a few good answers with detailed explanations, in Polynomial Long Division Confusion (simplifying $frac{x^{5}}{x^{2}+1}$). However, keep in mind that any values of $x$ which cause the denominator to be $0$ need to be handled appropriately.
$endgroup$
– John Omielan
Jan 13 at 2:07






$begingroup$
One thing to consider, since the numerator is a higher degree polynomial than the denominator, is to divide the denominator into the numerator. This is discussed, with a few good answers with detailed explanations, in Polynomial Long Division Confusion (simplifying $frac{x^{5}}{x^{2}+1}$). However, keep in mind that any values of $x$ which cause the denominator to be $0$ need to be handled appropriately.
$endgroup$
– John Omielan
Jan 13 at 2:07






1




1




$begingroup$
What is the result after dividing the numerator and demoninator by x - 1? Clearly 1 is not in the domain of y so calcuatling the nonexistent y(1) is TVidiotcy. At best it could be a value of a continuous extension of y.
$endgroup$
– William Elliot
Jan 13 at 3:33






$begingroup$
What is the result after dividing the numerator and demoninator by x - 1? Clearly 1 is not in the domain of y so calcuatling the nonexistent y(1) is TVidiotcy. At best it could be a value of a continuous extension of y.
$endgroup$
– William Elliot
Jan 13 at 3:33












2 Answers
2






active

oldest

votes


















2












$begingroup$

$$ frac{x^2 + 4x+11}{x+6} = k ; ; , ; ; x neq 1 $$



$$ x^2 + 4x+11 = kx+6k ; ; , ; ; x neq 1 $$
$$ x^2 + (4-k)x+(11-6k) = 0 ; ; , ; ; x neq 1 $$
This has (real) solutions $x$ when the discriminant is at least zero,
$$ (4-k)^2 - 4(11-6k) geq 0, $$
$$ k^2 - 8k+ 16 +24k - 44 geq 0, $$
$$ k^2 + 16k -28 geq 0, $$
$$ k^2 + 16k +64 geq 92, $$
$$ (k+8)^2 geq 92. $$
Either
$$ k+8 geq sqrt{92} $$
or
$$ k+8 leq -sqrt{92} $$
This last pair is the same information you would get from the quadratic formula applied to $k^2 + 16 k - 28.$



I don't recall what happened with the $x neq -1,$ you should draw some graphs



Note that
$$ y = frac{x^2 + 4x+11}{x+6} $$
describes a hyperbola, vertical asymptote at $x=-6,$ the other asymptote slanted (line $y=x-2$).






share|cite|improve this answer











$endgroup$





















    1












    $begingroup$

    We can write $$frac{x^3+3x^2+7x-11}{x^2+5x-6}=frac{x^2+4x+11}{x+6}=x-2+frac{23}{x+6},quad xne 1.
    $$
    If $x+6>0$, then by AM-GM inequality, we have
    $$
    x-2+frac{23}{x+6}=(x+6)+frac{23}{x+6}-8ge -8+2sqrt{23}.
    $$
    (Equality holds for $x+6=sqrt{23}$.) We can observe that the value of the function taken by $x+6=7$ is also attained by $x+6=frac{23}{7}$. Since it holds that $$lim_{xtoinfty}left(x-2+frac{23}{x+6}right)=lim_{xto -6^+}left(x-2+frac{23}{x+6}right)=infty,$$ the intermediate value theorem implies that the range of the given function on ${x+6>0, xne 1}$ is $$[-8+2sqrt{23},infty).$$



    On the other hand, if $x+6<0$, then
    $$
    (-x-6)+left(-frac{23}{x+6}right)ge 2sqrt{23}
    $$
    and
    $$
    (x+6)+frac{23}{x+6}-8le -8-2sqrt{23}.
    $$
    Since it holds $lim_{xto-infty}left(x-2+frac{23}{x+6}right)=-infty$, the range on ${x+6<0}$ is given by $$(-infty,-8-2sqrt{23}]$$ also by the intermediate value theorem.



    Gathering them, the range of the given function is
    $$
    (-infty,-8-2sqrt{23}]cup[-8+2sqrt{23},infty).
    $$






    share|cite|improve this answer











    $endgroup$













      Your Answer





      StackExchange.ifUsing("editor", function () {
      return StackExchange.using("mathjaxEditing", function () {
      StackExchange.MarkdownEditor.creationCallbacks.add(function (editor, postfix) {
      StackExchange.mathjaxEditing.prepareWmdForMathJax(editor, postfix, [["$", "$"], ["\\(","\\)"]]);
      });
      });
      }, "mathjax-editing");

      StackExchange.ready(function() {
      var channelOptions = {
      tags: "".split(" "),
      id: "69"
      };
      initTagRenderer("".split(" "), "".split(" "), channelOptions);

      StackExchange.using("externalEditor", function() {
      // Have to fire editor after snippets, if snippets enabled
      if (StackExchange.settings.snippets.snippetsEnabled) {
      StackExchange.using("snippets", function() {
      createEditor();
      });
      }
      else {
      createEditor();
      }
      });

      function createEditor() {
      StackExchange.prepareEditor({
      heartbeatType: 'answer',
      autoActivateHeartbeat: false,
      convertImagesToLinks: true,
      noModals: true,
      showLowRepImageUploadWarning: true,
      reputationToPostImages: 10,
      bindNavPrevention: true,
      postfix: "",
      imageUploader: {
      brandingHtml: "Powered by u003ca class="icon-imgur-white" href="https://imgur.com/"u003eu003c/au003e",
      contentPolicyHtml: "User contributions licensed under u003ca href="https://creativecommons.org/licenses/by-sa/3.0/"u003ecc by-sa 3.0 with attribution requiredu003c/au003e u003ca href="https://stackoverflow.com/legal/content-policy"u003e(content policy)u003c/au003e",
      allowUrls: true
      },
      noCode: true, onDemand: true,
      discardSelector: ".discard-answer"
      ,immediatelyShowMarkdownHelp:true
      });


      }
      });














      draft saved

      draft discarded


















      StackExchange.ready(
      function () {
      StackExchange.openid.initPostLogin('.new-post-login', 'https%3a%2f%2fmath.stackexchange.com%2fquestions%2f3071625%2frange-of-y-fracx33x27x-11x25x-6%23new-answer', 'question_page');
      }
      );

      Post as a guest















      Required, but never shown

























      2 Answers
      2






      active

      oldest

      votes








      2 Answers
      2






      active

      oldest

      votes









      active

      oldest

      votes






      active

      oldest

      votes









      2












      $begingroup$

      $$ frac{x^2 + 4x+11}{x+6} = k ; ; , ; ; x neq 1 $$



      $$ x^2 + 4x+11 = kx+6k ; ; , ; ; x neq 1 $$
      $$ x^2 + (4-k)x+(11-6k) = 0 ; ; , ; ; x neq 1 $$
      This has (real) solutions $x$ when the discriminant is at least zero,
      $$ (4-k)^2 - 4(11-6k) geq 0, $$
      $$ k^2 - 8k+ 16 +24k - 44 geq 0, $$
      $$ k^2 + 16k -28 geq 0, $$
      $$ k^2 + 16k +64 geq 92, $$
      $$ (k+8)^2 geq 92. $$
      Either
      $$ k+8 geq sqrt{92} $$
      or
      $$ k+8 leq -sqrt{92} $$
      This last pair is the same information you would get from the quadratic formula applied to $k^2 + 16 k - 28.$



      I don't recall what happened with the $x neq -1,$ you should draw some graphs



      Note that
      $$ y = frac{x^2 + 4x+11}{x+6} $$
      describes a hyperbola, vertical asymptote at $x=-6,$ the other asymptote slanted (line $y=x-2$).






      share|cite|improve this answer











      $endgroup$


















        2












        $begingroup$

        $$ frac{x^2 + 4x+11}{x+6} = k ; ; , ; ; x neq 1 $$



        $$ x^2 + 4x+11 = kx+6k ; ; , ; ; x neq 1 $$
        $$ x^2 + (4-k)x+(11-6k) = 0 ; ; , ; ; x neq 1 $$
        This has (real) solutions $x$ when the discriminant is at least zero,
        $$ (4-k)^2 - 4(11-6k) geq 0, $$
        $$ k^2 - 8k+ 16 +24k - 44 geq 0, $$
        $$ k^2 + 16k -28 geq 0, $$
        $$ k^2 + 16k +64 geq 92, $$
        $$ (k+8)^2 geq 92. $$
        Either
        $$ k+8 geq sqrt{92} $$
        or
        $$ k+8 leq -sqrt{92} $$
        This last pair is the same information you would get from the quadratic formula applied to $k^2 + 16 k - 28.$



        I don't recall what happened with the $x neq -1,$ you should draw some graphs



        Note that
        $$ y = frac{x^2 + 4x+11}{x+6} $$
        describes a hyperbola, vertical asymptote at $x=-6,$ the other asymptote slanted (line $y=x-2$).






        share|cite|improve this answer











        $endgroup$
















          2












          2








          2





          $begingroup$

          $$ frac{x^2 + 4x+11}{x+6} = k ; ; , ; ; x neq 1 $$



          $$ x^2 + 4x+11 = kx+6k ; ; , ; ; x neq 1 $$
          $$ x^2 + (4-k)x+(11-6k) = 0 ; ; , ; ; x neq 1 $$
          This has (real) solutions $x$ when the discriminant is at least zero,
          $$ (4-k)^2 - 4(11-6k) geq 0, $$
          $$ k^2 - 8k+ 16 +24k - 44 geq 0, $$
          $$ k^2 + 16k -28 geq 0, $$
          $$ k^2 + 16k +64 geq 92, $$
          $$ (k+8)^2 geq 92. $$
          Either
          $$ k+8 geq sqrt{92} $$
          or
          $$ k+8 leq -sqrt{92} $$
          This last pair is the same information you would get from the quadratic formula applied to $k^2 + 16 k - 28.$



          I don't recall what happened with the $x neq -1,$ you should draw some graphs



          Note that
          $$ y = frac{x^2 + 4x+11}{x+6} $$
          describes a hyperbola, vertical asymptote at $x=-6,$ the other asymptote slanted (line $y=x-2$).






          share|cite|improve this answer











          $endgroup$



          $$ frac{x^2 + 4x+11}{x+6} = k ; ; , ; ; x neq 1 $$



          $$ x^2 + 4x+11 = kx+6k ; ; , ; ; x neq 1 $$
          $$ x^2 + (4-k)x+(11-6k) = 0 ; ; , ; ; x neq 1 $$
          This has (real) solutions $x$ when the discriminant is at least zero,
          $$ (4-k)^2 - 4(11-6k) geq 0, $$
          $$ k^2 - 8k+ 16 +24k - 44 geq 0, $$
          $$ k^2 + 16k -28 geq 0, $$
          $$ k^2 + 16k +64 geq 92, $$
          $$ (k+8)^2 geq 92. $$
          Either
          $$ k+8 geq sqrt{92} $$
          or
          $$ k+8 leq -sqrt{92} $$
          This last pair is the same information you would get from the quadratic formula applied to $k^2 + 16 k - 28.$



          I don't recall what happened with the $x neq -1,$ you should draw some graphs



          Note that
          $$ y = frac{x^2 + 4x+11}{x+6} $$
          describes a hyperbola, vertical asymptote at $x=-6,$ the other asymptote slanted (line $y=x-2$).







          share|cite|improve this answer














          share|cite|improve this answer



          share|cite|improve this answer








          edited Jan 13 at 3:35

























          answered Jan 13 at 3:25









          Will JagyWill Jagy

          103k5101200




          103k5101200























              1












              $begingroup$

              We can write $$frac{x^3+3x^2+7x-11}{x^2+5x-6}=frac{x^2+4x+11}{x+6}=x-2+frac{23}{x+6},quad xne 1.
              $$
              If $x+6>0$, then by AM-GM inequality, we have
              $$
              x-2+frac{23}{x+6}=(x+6)+frac{23}{x+6}-8ge -8+2sqrt{23}.
              $$
              (Equality holds for $x+6=sqrt{23}$.) We can observe that the value of the function taken by $x+6=7$ is also attained by $x+6=frac{23}{7}$. Since it holds that $$lim_{xtoinfty}left(x-2+frac{23}{x+6}right)=lim_{xto -6^+}left(x-2+frac{23}{x+6}right)=infty,$$ the intermediate value theorem implies that the range of the given function on ${x+6>0, xne 1}$ is $$[-8+2sqrt{23},infty).$$



              On the other hand, if $x+6<0$, then
              $$
              (-x-6)+left(-frac{23}{x+6}right)ge 2sqrt{23}
              $$
              and
              $$
              (x+6)+frac{23}{x+6}-8le -8-2sqrt{23}.
              $$
              Since it holds $lim_{xto-infty}left(x-2+frac{23}{x+6}right)=-infty$, the range on ${x+6<0}$ is given by $$(-infty,-8-2sqrt{23}]$$ also by the intermediate value theorem.



              Gathering them, the range of the given function is
              $$
              (-infty,-8-2sqrt{23}]cup[-8+2sqrt{23},infty).
              $$






              share|cite|improve this answer











              $endgroup$


















                1












                $begingroup$

                We can write $$frac{x^3+3x^2+7x-11}{x^2+5x-6}=frac{x^2+4x+11}{x+6}=x-2+frac{23}{x+6},quad xne 1.
                $$
                If $x+6>0$, then by AM-GM inequality, we have
                $$
                x-2+frac{23}{x+6}=(x+6)+frac{23}{x+6}-8ge -8+2sqrt{23}.
                $$
                (Equality holds for $x+6=sqrt{23}$.) We can observe that the value of the function taken by $x+6=7$ is also attained by $x+6=frac{23}{7}$. Since it holds that $$lim_{xtoinfty}left(x-2+frac{23}{x+6}right)=lim_{xto -6^+}left(x-2+frac{23}{x+6}right)=infty,$$ the intermediate value theorem implies that the range of the given function on ${x+6>0, xne 1}$ is $$[-8+2sqrt{23},infty).$$



                On the other hand, if $x+6<0$, then
                $$
                (-x-6)+left(-frac{23}{x+6}right)ge 2sqrt{23}
                $$
                and
                $$
                (x+6)+frac{23}{x+6}-8le -8-2sqrt{23}.
                $$
                Since it holds $lim_{xto-infty}left(x-2+frac{23}{x+6}right)=-infty$, the range on ${x+6<0}$ is given by $$(-infty,-8-2sqrt{23}]$$ also by the intermediate value theorem.



                Gathering them, the range of the given function is
                $$
                (-infty,-8-2sqrt{23}]cup[-8+2sqrt{23},infty).
                $$






                share|cite|improve this answer











                $endgroup$
















                  1












                  1








                  1





                  $begingroup$

                  We can write $$frac{x^3+3x^2+7x-11}{x^2+5x-6}=frac{x^2+4x+11}{x+6}=x-2+frac{23}{x+6},quad xne 1.
                  $$
                  If $x+6>0$, then by AM-GM inequality, we have
                  $$
                  x-2+frac{23}{x+6}=(x+6)+frac{23}{x+6}-8ge -8+2sqrt{23}.
                  $$
                  (Equality holds for $x+6=sqrt{23}$.) We can observe that the value of the function taken by $x+6=7$ is also attained by $x+6=frac{23}{7}$. Since it holds that $$lim_{xtoinfty}left(x-2+frac{23}{x+6}right)=lim_{xto -6^+}left(x-2+frac{23}{x+6}right)=infty,$$ the intermediate value theorem implies that the range of the given function on ${x+6>0, xne 1}$ is $$[-8+2sqrt{23},infty).$$



                  On the other hand, if $x+6<0$, then
                  $$
                  (-x-6)+left(-frac{23}{x+6}right)ge 2sqrt{23}
                  $$
                  and
                  $$
                  (x+6)+frac{23}{x+6}-8le -8-2sqrt{23}.
                  $$
                  Since it holds $lim_{xto-infty}left(x-2+frac{23}{x+6}right)=-infty$, the range on ${x+6<0}$ is given by $$(-infty,-8-2sqrt{23}]$$ also by the intermediate value theorem.



                  Gathering them, the range of the given function is
                  $$
                  (-infty,-8-2sqrt{23}]cup[-8+2sqrt{23},infty).
                  $$






                  share|cite|improve this answer











                  $endgroup$



                  We can write $$frac{x^3+3x^2+7x-11}{x^2+5x-6}=frac{x^2+4x+11}{x+6}=x-2+frac{23}{x+6},quad xne 1.
                  $$
                  If $x+6>0$, then by AM-GM inequality, we have
                  $$
                  x-2+frac{23}{x+6}=(x+6)+frac{23}{x+6}-8ge -8+2sqrt{23}.
                  $$
                  (Equality holds for $x+6=sqrt{23}$.) We can observe that the value of the function taken by $x+6=7$ is also attained by $x+6=frac{23}{7}$. Since it holds that $$lim_{xtoinfty}left(x-2+frac{23}{x+6}right)=lim_{xto -6^+}left(x-2+frac{23}{x+6}right)=infty,$$ the intermediate value theorem implies that the range of the given function on ${x+6>0, xne 1}$ is $$[-8+2sqrt{23},infty).$$



                  On the other hand, if $x+6<0$, then
                  $$
                  (-x-6)+left(-frac{23}{x+6}right)ge 2sqrt{23}
                  $$
                  and
                  $$
                  (x+6)+frac{23}{x+6}-8le -8-2sqrt{23}.
                  $$
                  Since it holds $lim_{xto-infty}left(x-2+frac{23}{x+6}right)=-infty$, the range on ${x+6<0}$ is given by $$(-infty,-8-2sqrt{23}]$$ also by the intermediate value theorem.



                  Gathering them, the range of the given function is
                  $$
                  (-infty,-8-2sqrt{23}]cup[-8+2sqrt{23},infty).
                  $$







                  share|cite|improve this answer














                  share|cite|improve this answer



                  share|cite|improve this answer








                  edited Jan 13 at 6:46

























                  answered Jan 13 at 6:17









                  SongSong

                  12.6k631




                  12.6k631






























                      draft saved

                      draft discarded




















































                      Thanks for contributing an answer to Mathematics Stack Exchange!


                      • Please be sure to answer the question. Provide details and share your research!

                      But avoid



                      • Asking for help, clarification, or responding to other answers.

                      • Making statements based on opinion; back them up with references or personal experience.


                      Use MathJax to format equations. MathJax reference.


                      To learn more, see our tips on writing great answers.




                      draft saved


                      draft discarded














                      StackExchange.ready(
                      function () {
                      StackExchange.openid.initPostLogin('.new-post-login', 'https%3a%2f%2fmath.stackexchange.com%2fquestions%2f3071625%2frange-of-y-fracx33x27x-11x25x-6%23new-answer', 'question_page');
                      }
                      );

                      Post as a guest















                      Required, but never shown





















































                      Required, but never shown














                      Required, but never shown












                      Required, but never shown







                      Required, but never shown

































                      Required, but never shown














                      Required, but never shown












                      Required, but never shown







                      Required, but never shown







                      Popular posts from this blog

                      Can a sorcerer learn a 5th-level spell early by creating spell slots using the Font of Magic feature?

                      Does disintegrating a polymorphed enemy still kill it after the 2018 errata?

                      A Topological Invariant for $pi_3(U(n))$